Few businesses will

This topic has expert replies
Legendary Member
Posts: 1112
Joined: Sat Feb 26, 2011 11:16 am
Thanked: 77 times
Followed by:49 members

Few businesses will

by atulmangal » Fri Jun 03, 2011 10:37 am
Princeton Review

Few businesses will voluntarily implement environmental protection measures that benefit the public if those measures reduce profitability. Even though such measures may cost less to implement than the total value of their benefits to society, the company making the decision bears all of the cost and receives little, if any, of the benefit. For example, XYZ Corporation has for the last 10 years refused to install smokestack filters to reduce the air pollution emitted by its factory, claiming that the cost would be prohibitive. Therefore, if such measures are to be implemented to protect the environment, they must be initiated by government regulation or intervention.

The bolded portions of the argument above perform which of the following functions?


A. The first phrase states the conclusion, and the second provides evidence, the truth of which supports the validity of the conclusion.

B. The first phrase states a premise supporting the conclusion, and the second provides evidence, the falsity of which would disprove the first phrase.

C. The first phrase describes a general principle, and the second provides evidence countering that principle.

D. The first phrase states a claim supporting the conclusion, but for which no evidence is given, and the second describes an example supporting the conclusion.

E. The first phrase states a generalization supporting the conclusion, and the second cites an example supporting that generalization.
Last edited by atulmangal on Fri Jun 03, 2011 11:05 am, edited 1 time in total.

User avatar
Legendary Member
Posts: 1309
Joined: Mon Apr 04, 2011 5:34 am
Location: India
Thanked: 310 times
Followed by:123 members
GMAT Score:750

by cans » Fri Jun 03, 2011 10:54 am
Hey you forgot to bold the lines..
If my post helped you- let me know by pushing the thanks button ;)

Contact me about long distance tutoring!
[email protected]

Cans!!

Legendary Member
Posts: 1112
Joined: Sat Feb 26, 2011 11:16 am
Thanked: 77 times
Followed by:49 members

by atulmangal » Fri Jun 03, 2011 11:05 am
cans wrote:Hey you forgot to bold the lines..
Thanks edited the post...

User avatar
Master | Next Rank: 500 Posts
Posts: 407
Joined: Tue Jan 25, 2011 9:19 am
Thanked: 25 times
Followed by:7 members

by Ozlemg » Fri Jun 03, 2011 12:04 pm
I am btw B and D
IMO D

I easily eliminate A.
For C, D, first parts seem correct but second parts are wrong.
For B, first part is a claim more than a premise.
The more you suffer before the test, the less you will do so in the test! :)

Junior | Next Rank: 30 Posts
Posts: 29
Joined: Sun Feb 20, 2011 11:03 pm

by coolly01 » Fri Jun 03, 2011 6:33 pm
Between B and E,
But I pick E

Master | Next Rank: 500 Posts
Posts: 112
Joined: Sun Nov 08, 2009 9:39 pm
Location: Delhi
Thanked: 2 times

by dv2020 » Sat Jun 04, 2011 8:07 pm
between [spoiler]B, D & E[/spoiler] IMO E

all the choices talk about first stmt supporting conclusion. B says if 2nd stmt is not there ist would be refuted. 2nd is a example not the only reason B could be true. in D it says there is no ex of the 1st stmt. In E indeed 2nd acts as an evidence for 1st. Hence E

User avatar
Legendary Member
Posts: 1309
Joined: Mon Apr 04, 2011 5:34 am
Location: India
Thanked: 310 times
Followed by:123 members
GMAT Score:750

by cans » Sat Jun 04, 2011 9:26 pm
IMO B
If my post helped you- let me know by pushing the thanks button ;)

Contact me about long distance tutoring!
[email protected]

Cans!!

Master | Next Rank: 500 Posts
Posts: 110
Joined: Tue Dec 23, 2008 4:52 am
Thanked: 3 times

by M09 » Sun Jun 05, 2011 5:48 am
atulmangal wrote:Princeton Review

Few businesses will voluntarily implement environmental protection measures that benefit the public if those measures reduce profitability. Even though such measures may cost less to implement than the total value of their benefits to society, the company making the decision bears all of the cost and receives little, if any, of the benefit. For example, XYZ Corporation has for the last 10 years refused to install smokestack filters to reduce the air pollution emitted by its factory, claiming that the cost would be prohibitive. Therefore, if such measures are to be implemented to protect the environment, they must be initiated by government regulation or intervention.

The bolded portions of the argument above perform which of the following functions?


A. The first phrase states the conclusion, and the second provides evidence, the truth of which supports the validity of the conclusion.

B. The first phrase states a premise supporting the conclusion, and the second provides evidence, the falsity of which would disprove the first phrase.

C. The first phrase describes a general principle, and the second provides evidence countering that principle.

D. The first phrase states a claim supporting the conclusion, but for which no evidence is given, and the second describes an example supporting the conclusion.

E. The first phrase states a generalization supporting the conclusion, and the second cites an example supporting that generalization.
#1 : Bold stat 1 and #2 : Bold stat 2
Between B, D & E
Rejected B - the second half of the option states #2 is evidence..I feel it's an example
Rejected E- #1 is not a generalization
IMO D

Legendary Member
Posts: 1112
Joined: Sat Feb 26, 2011 11:16 am
Thanked: 77 times
Followed by:49 members

by atulmangal » Sun Jun 05, 2011 6:28 am
Hey Guys, Surprise...OA is E

Reasoning given for Op E is:

The first phrase is a general principle that supports the conclusion that government intervention is necessary to get businesses to implement environmental protection measures. The second phrase is an example of what the first phrase describes (businesses won't do it on their own). The correct answer is E

User avatar
Legendary Member
Posts: 1325
Joined: Sun Nov 01, 2009 6:24 am
Thanked: 105 times
Followed by:14 members

by vikram4689 » Sun Jun 05, 2011 6:54 am
I picked E and was confused as i was scrolling down ;) to see so many varied responses,which is something that is not seen often.

A-1st is NOT conclusion
B-Falsity of 2nd wont disprove 1st
C-2nd does not counteract 1st
D-Evidence is given
E-Correct

Nice Question
Premise: If you like my post
Conclusion : Press the Thanks Button ;)

User avatar
Senior | Next Rank: 100 Posts
Posts: 99
Joined: Sat Oct 16, 2010 1:55 pm
Thanked: 1 times

by aftableo2006 » Tue Jun 07, 2011 12:47 am
good question my take is B